Finished De Bruijn sections
This commit is contained in:
@ -97,5 +97,8 @@ Construct $\mathcal{L}(G_2)$ and $\mathcal{L}(G_3)$. What do you notice?
|
||||
|
||||
\begin{solution}
|
||||
After fixing edge labels, we find that
|
||||
$\mathcal{L}(G_2) \cong G_3$, and $\mathcal{L}(G_3) \cong G_4$
|
||||
\end{solution}
|
||||
$\mathcal{L}(G_2) \cong G_3$ and $\mathcal{L}(G_3) \cong G_4$
|
||||
\end{solution}
|
||||
|
||||
\vfill
|
||||
\pagebreak
|
Reference in New Issue
Block a user